Question 16

Which of the following, if true, would most weaken the argument presented in the previous question?

Solution

Option A: Since this option is irrelevant to the author's argument, this is not the correct option.

Options B and D: This option strengthens the author's argument rather than weakening it. Thus, this is not the correct option.

Option E: If the internet's usage enhances the youth's critical thinking, then this weakens the author's argument about the harmful effects of the internet on people.

Thus, the correct option is E.


Create a FREE account and get:

  • All Quant Formulas and shortcuts PDF
  • XAT previous papers with solutions PDF
  • XAT Trial Classes for FREE

    cracku

    Boost your Prep!

    Download App